petit problème
Répondre à la discussion
Page 1 sur 2 1 DernièreDernière
Affichage des résultats 1 à 30 sur 31

petit problème



  1. #1
    matthias

    petit problème


    ------

    Je cherche trois entiers naturels non nuls a, b et c qui vérifient :









    Pouvez vous m'aider ?

    -----

  2. #2
    invitebdaccd77

    Re : petit problème

    Pour réaliser les 3 premières containtes c'est facile:

    voici 10 possibilités

    44 117 240
    85 132 720
    88 234 480
    132 351 720
    140 480 693
    160 231 792
    176 468 960
    240 252 275
    480 504 550
    720 756 825

    Pour la quatrième ça se complique. J'ai rien trouvé numériquement jusqu'a 1000 pour a,b et c

  3. #3
    matthias

    Re : petit problème

    oui pour les trois premières, tout va bien

  4. #4
    matthias

    Re : petit problème

    Ah oui au fait, ça correspond à un parallélépipède dont les côtés et toutes les diagonales sont des entiers.

  5. A voir en vidéo sur Futura
  6. #5
    invite43f8e83d

    Re : petit problème

    bonjour, Matthias, il semble que la résolution du Pb demandé ne soit pas possible.
    indice: si a²+b²=d² et b²+c²=e² et a²+c²=f²
    à quoi est égale la quantité a²+b²+c²?
    à quoi est égale la quantité racine de (a²+b²+c²)?
    est-ce que celà peut être un entier si (a²+b²+c²) est un entier?

  7. #6
    matthias

    Re : petit problème

    Citation Envoyé par duduc
    bonjour, Matthias, il semble que la résolution du Pb demandé ne soit pas possible.
    indice: si a²+b²=d² et b²+c²=e² et a²+c²=f²
    à quoi est égale la quantité a²+b²+c²?
    à quoi est égale la quantité racine de (a²+b²+c²)?
    est-ce que celà peut être un entier si (a²+b²+c²) est un entier?
    D'abord, je pense aussi que le problème est impossible, mais tes arguments ne me permettent pas de conclure, en tous cas pas de manière immédiate.

    si ce que tu veux dire est que

    et de dire que ne peut pas être un entier, alors je ne suis pas d'accord.



    Cela dit cet argument donne des restrictions naturelles sur a, b et c, et permet peut-être de conclure simplement, mais en ce cas quelque chose m'échappe. Tu peux développer tes explications stp ?

  8. #7
    g_h

    Re : petit problème

    J'ai testé pour tous les entiers jusqu'à 10000, et je n'ai rien trouvé non plus

  9. #8
    inviteca3a9be7

    Re : petit problème

    Salut,


    Un tel parallélépipède est appelé "Cube parfait" (un cas particulier de la "Brique parfaite d’Euler"). On n'en connait aucun exemple !!! C'est un problème ouvert d'en trouver un ou de prouver qu'il n'y en a pas.

    cf http://mathworld.wolfram.com/EulerBrick.html


    Hope ça helps

  10. #9
    matthias

    Re : petit problème

    mouarf, je sais
    bon, j'ai été un peu vache de pas le dire avant, mais je n'avais trouvé qu'un seul site en parlant (je n'avais pas cherché en anglais), et comme je trouvais incroyable que ce problème soit si peu connu, je me demandais si ce n'était pas une blague.
    Je m'étonne quand même que les grecs ne se soient pas intérréssé au problème (ou qu'on ne le sache pas). Un parallélépipède dont les côtés et les diagonales sont entières, ça ressemble vraiment à un problème antique, vous ne trouvez pas ?

    Sinon, sans chercher à résoudre un problème où de bien meilleurs que moi se sont cassé les dents, on doit quand même pouvoir donner des conditions assez précises sur a, b et c, non ?

    En passant, , pourquoi un cas particulier de la brique parfaite, il y a un problème plus général ? Le cas que j'ai donné ne correspond pas particulièrement à un cube. Et merci pour le lien, je vais voir ça.

  11. #10
    leg

    Re : petit problème

    Bonjour matthias,
    je pensais que depuis la résolution du dernier théorème de Fermat , ce problème était résolu…
    Car si je comprends bien la question :
    Si : a² + b² = d² ; a² + c² = e² ; b² + c² = f²
    Ainsi que a² +b² + c² = u² puisque la racine carrée de : a² +b² + c² doit être un entier naturel positif, cela équivaut à dire :
    d² + c² = u² , e² + b² = u² ainsi que f² + a² = u² ! soit trois hypoténuses identiques pour trois Triplets Pythagoricien différents.. ? avec par obligation trois couples d’entiers ou non > 0, différents, qui sont les paramètres de ces trois triangles rectangles ce qui est impossible, il n’existe aucun cas dans les Triplets Pythagoriciens… ! la réponse me parait donc évidente a moins d’une erreur ..
    (p² + q²)² = d² ; p et q sont les paramètres du T1 , d étant l’hypoténuse
    (u² + v²)² = e² u et v ….. de T2 , e étant l’hypoténuse
    (i² + h² )² = f² i et h ….. de T3 , f étant l’hypoténuse
    Comment trois couples de paramètres différent pourrait donner le même Hypothénuse ?
    Si une telle solution existait :
    a) est ce que le théorème de Pythagore voudrait encore dire quelque chose ? le carré de l’ hypothénuse = la somme des carrés des côtés de l’angle droit…. ?
    b) cela pourrait-il remètre en question le dernier théorème de Fermat, si un cube parfait existait, il existerait probablement une solution dans la puissance 6 tel que: X^6 + Y^6 = Z^6 ce qui est impossible!
    A plus leg.

  12. #11
    matthias

    Re : petit problème

    Citation Envoyé par leg
    Bonjour matthias,
    je pensais que depuis la résolution du dernier théorème de Fermat , ce problème était résolu…
    Peut-être que le théorème de Fermat permet de résoudre ce problème, mais a priori personne n'a encore trouvé la solution.

    Citation Envoyé par leg
    Car si je comprends bien la question :
    Si : a² + b² = d² ; a² + c² = e² ; b² + c² = f²
    Ainsi que a² +b² + c² = u² puisque la racine carrée de : a² +b² + c² doit être un entier naturel positif, cela équivaut à dire :
    d² + c² = u² , e² + b² = u² ainsi que f² + a² = u² ! soit trois hypoténuses identiques pour trois Triplets Pythagoricien différents.. ? avec par obligation trois couples d’entiers ou non > 0, différents, qui sont les paramètres de ces trois triangles rectangles ce qui est impossible, il n’existe aucun cas dans les Triplets Pythagoriciens… !
    Ah non ? Ca m'étonne mais je vais regarder.

    Citation Envoyé par leg
    la réponse me parait donc évidente a moins d’une erreur ..
    (p² + q²)² = d² ; p et q sont les paramètres du T1 , d étant l’hypoténuse
    (u² + v²)² = e² u et v ….. de T2 , e étant l’hypoténuse
    (i² + h² )² = f² i et h ….. de T3 , f étant l’hypoténuse
    Comment trois couples de paramètres différent pourrait donner le même Hypothénuse ?
    Excuse moi, j'ai du mal à suivre. C'est quoi p,q,u,v,i,h ?
    Quand tu parles de paramètre d'un triplet pythagoricien, tu parles bien des nombres u et v suivants ?

    a,b et c premiers entre eux, b pair




    Si ça n'est pas ça, alors explique moi de quoi tu parles s'il te plait.


    Citation Envoyé par leg
    Si une telle solution existait :
    a) est ce que le théorème de Pythagore voudrait encore dire quelque chose ? le carré de l’ hypothénuse = la somme des carrés des côtés de l’angle droit…. ?
    b) cela pourrait-il remètre en question le dernier théorème de Fermat, si un cube parfait existait, il existerait probablement une solution dans la puissance 6 tel que: X^6 + Y^6 = Z^6 ce qui est impossible!
    A plus leg.
    Je n'ai pas de réponse à ces questions, ça ne me paraît pas évident.

  13. #12
    leg

    Re : petit problème

    oui matthias, c'est bien les paramètres u et v pour construire par ex le triplet Pyth de a,b et c premiers entre eux, b pair.
    donc p et q sont les paramètres du triangle 1, ou d est l'ypothénuse, ..etc avec u et v et ainsi que i et h pour contruire T3.
    ma question est donc : est ce que l'on a déjà trouvé trois triangles rectangles différents, donné par trois couples de paramètres différents mais ayant la même hypothénuse...? cela correspond a ta supposition :racine carréede a² + b²+c² doit être un entier, d'où cette contradiction 3 triangles ayant le même hypothénuse:
    d² + c²= u², e²+ b² = u² et f² +a²= u² puisque a² + b²= d²,étant donné que la racine carrée doit être un entier! ainsi que pour a²+c² et b² + c².
    A + matt, leg

  14. #13
    leg

    Re : petit problème

    Citation Envoyé par leg
    oui matthias, c'est bien les paramètres u et v pour construire par ex le triplet Pyth de a,b et c premiers entre eux, b pair.
    3 triangles ayant le même hypothénuse:
    d² + c²= u², e²+ b² = u² et f² +a²= u² puisque a² + b²= d²,étant donné que la racine carrée doit être un entier! ainsi que pour a²+c² et b² + c².
    A + matt, leg
    bonjour matthias
    je reviens sur ta question et sur mes suppositions, voici ce que j'en ai déduit:
    .............................. .............................. ................
    petit rappel
    si je comprends bien la question :
    Si : a² + b² = d² ; a² + c² = e² ; b² + c² = f²
    Ainsi que a² +b² + c² = u² puisque la racine carrée de : a² +b² + c² doit être un entier naturel positif, cela équivaut à dire :
    d² + c² = u² , e² + b² = u² ainsi que f² + a² = u² !

    soit trois hypoténuses identiques pour trois Triplets Pythagoricien différents.. ?
    .............................. .............................. .................
    Il est facile de voir, que a²+b²+ c² =u² implique obligatoirement un triangle t2:
    D²+ c² = u² avec D² = a²+b², donc D qui est l’hypoténuse du premier T.P (triplet pythagoricien) T1, quelconque, redonne p² - q²avec p et q > p’ et q’ ayant donné D !
    Par exemple 5 = p’²+q’², puis 5 = p² - q², etc etc on peut donc dire que c , est pair ainsi que b d’ailleurs, et c, serra obligatoirement fixé en fonction de D, il faudra à nouveau un a’ tel que:
    a’ = u²-v² avec a’ = a mais avec u et v différent de p’ et q’ ayant donné a , b et D et où c = 2uv mais aussi 2pq. Ayant trouvé facilement D et c il faut cherché a’ et c.
    Tel que a’² + c²= E² de sorte que E² + b² = u²
    A nouveau, E va déterminé b , or b est déjà connu !
    Alors il apparaît une contradiction simple qui reste à analyser si besoin est :
    Nous voyons que les paramètres p et q sont > p’ et q’ et donc que c > b.
    En effet t3 est donné par : u² - v² =a’ =a ; 2uv = c et bien sur u² + v² = E, l’hypoténuse; il existe bien t4 donné par m et w tel que m² - w² = E mais m et w sont > u et v donc 2mw = b > c ce qui est contradictoire avec t2 où c > b !
    La question posée n’a donc pas de solution !

    quel est ton avis ?

  15. #14
    matthias

    Re : petit problème

    Je ne comprends pas tes arguments de parité. Tous les triplets ne sont pas nécessairement primitifs.

  16. #15
    leg

    Re : petit problème

    Citation Envoyé par matthias
    Je ne comprends pas tes arguments de parité. Tous les triplets ne sont pas nécessairement primitifs.
    c'est exact, mais quelque soit le triplet choisis pour obtenir a² +b² +c² = u² il faut bien a chaque fois repartir de l'hypoténuse!
    a²+b² = d² et d² +c² =u²
    a² +c² = e² et e² + b² =u²
    b²+c² = f² tu peux constaté d'ailleur que ce triplet, ne peut en aucun cas être un triplet primitif car b et c sont pair ce qui implique f pair donc il existe un primitif inferieur . Cela en supposant que je suis parti d'un triplet a, b et d avec b pair et a et d, impair.

    donc cn'est pas arbitraire il est déterminé par d afin de répondre à l'égalité d²+c² = u²
    il en serra de même pour b et pourtant au départ je peux partir de n'importe quel triplet pythagoricien; en choisissant a qui me donne b puis d, mais je revien sur b ensuite qui doit répondre à l'égalité a² +c² = e² tel qu'ensuite e²+b² = u²!
    tu peux aussi constater qu'en dessinant les triangles rectangles et en redéssinant un triangle rectangle sur l'hypoténuse, tu obtient un escargot qui part des plus petits entiers et tu tourne vers l'infiniment grand mais pas l'inverse.
    exemple3 , 4 et 5 puis 5 , 12 et 13; puis 13 , 84 et 85 etc ..mais cela ne change rien si je part par exemple de
    6 , 8 et 10 puis 10, 24 et 26 etc...
    le problème n'est pas dans la parité, mais dans les question de grandeur c est superieur a b dans t2 > t1 donc b ne peut en aucun cas redevenir > c dans t3 et t4 ce qui serait absurde.
    regarde bien les paramètres de la formule qui distribue les triplets,
    et ensuite tu termines avec b² et c² =f² qui donne f² +a² = u² où; a , devrait être superieur à tous ces nombres, alors qu'au départ c'est le plus petit entier par obligation, quelque soit le triplet de départ.
    je ne peux pas tourner à l'envers avec des entiers de plus en plus petits...!
    ta question et ta contrainte impose des paramètres de plus en plus grand, mais, en revenant sur les nombres qui leur sont inferieur! c'est la contradiction recherché il me semble...

  17. #16
    matthias

    Re : petit problème

    Bon je suis d'accord avec le fait que l'on peut imposer sans perte de généralité a impair, b pair, c pair.
    On a aussi d = p'² + q'² = p² - q², donc p > p' et p > q'.

    Mais après je ne suis plus.

  18. #17
    invite35452583

    Re : petit problème

    Citation Envoyé par leg
    ma question est donc : est ce que l'on a déjà trouvé trois triangles rectangles différents, donné par trois couples de paramètres différents mais ayant la même hypothénuse...?
    Oui, exemple
    65²=4225=60²+25²=52²+39²=56²+3 3²
    Tout simplement trouvé ainsi (N : norme des entiers de Gauss)
    5=N(1+2i)
    13=N(2+3i)
    Et on permutte
    N[(1+2i)(1-2i)(2+3i)(2-3i)] donne 5².13²=(5.13)²
    N[(1+2i)(1+2i)(2+3i)(2-3i)] donne (3²+4²).13²=(3.13)²+(4.13)²
    N[(1+2i)(1-2i)(2+3i)(2+3i)] donne 5².(2²+3²)=(5.2)²+(5.3)²
    N[(1+2i)(1+2i)(2+3i)(2+3i)] donne un "nouveau" N(33+56i)=33²+56²

    Ainsi en utilisant les entiers de Gauss, on montre assez facilement (un peu long à écrire) :
    1) il existe une infinité de triplets de triangles pythagoriciens ayant même hypothénuse
    2) on a même : pour entier k, on peut trouver un carré qui est l'hypothénuse d'au moins k triangles pythagoriciens.


    Citation Envoyé par leg
    ta question et ta contrainte impose des paramètres de plus en plus grand, mais, en revenant sur les nombres qui leur sont inferieur! c'est la contradiction recherché il me semble...
    Reprenons avec 65
    65=33²-32² ce qui donne 65²+2012²+2113²
    mais on a aussi 65=9²-4² ce qui donne 65²+72²=97²
    Les paramètres toujours plus grands ? Les paramètres de 65²=56²+33² sont 7 et 4 (donc globalement plus petits que 9 et 4)

    J'ai l'impression que tu as oublié que les paramètres pour un nombre donné ne sont pas uniques contrairement aux triplets. Le coup de l'escargot fonctionne sur tes exemples parce que 5 et 13 sont premiers.
    Quand on cherche à paramétrer, on a :
    5 (ou 13)=p²-q²=(p+q)(p-q) ce qui laisse l'unique possibilité (avec p>q>0) p+q=5 (ou 13) et p-q=1. Mais ce n'est plus le cas quand le nombre n'est plus premier (cas côté qui n'est pas l'hypothénuse) pour p²-q², et n'est plus le cas pour (cas côté=hypothénuse) p²+q² quand p+iq n'est pas premier dans Z[i].

  19. #18
    invite35452583

    Re : petit problème

    [et de dire que ne peut pas être un entier, alors je ne suis pas d'accord.


    [/QUOTE]
    Il existe même une famille infinie "évidente" de solutions, tout simplement celles du type :

  20. #19
    leg

    Re : petit problème

    Citation Envoyé par matthias
    Bon je suis d'accord avec le fait que l'on peut imposer sans perte de généralité a impair, b pair, c pair.
    On a aussi d = p'² + q'² = p² - q², donc p > p' et p > q'.

    Mais après je ne suis plus.
    bonjour Matthias
    lorsque l'on repart de d qui n'est plus l'hypoténuse mais le côté du triangle rect t2, tu est d'accord que t2 > t1 et aussi que les nouveaux paramètres de t2 aussi, donc p > p' et p > q'; le deuxième côté de t2 est obligatoirement cselon l'hypthèse de départ, puisque d² + c² = u²;
    ("cn'est donc pas arbitraire dans le choix des trois entiers a, b et c.")
    tu en conviendras que 2p'q' = b est < 2pq = c.

    il te faut donc, recommencer la même opération mais avec a , c et E tel que a²+ c² = E² = t3, de sorte que E² + b² = u² suivant la supposition de départ:
    [a² + c² + b² = u²] = [ E² + b² ] = t4; tu est donc confonté à la même solution mais avec :
    1) b est déterminé par E
    2) les paramètres deviennent p''² + q''² = E et bien sur p'''² - q'''² = E avec (comme pour t1 et t2) p''< p''' et q'' < q''' d'où t3 < t4 (on peu d'ailleur remarquer que t1 <t2 <t3 <t4) ce qui nous importe, c'est que b dans t4 est obligatoirement > cpuisque 2p'' q'' < 2p''' q''' donc c < b ce qui est absurde il était > b dans t1 et t2...!
    d'où d'aprés moi la contradiction recherchée,

  21. #20
    leg

    Re : petit problème

    suite:
    en plus lorsque l'on refait de mêmeavec le dernier triplet
    t5 et t6
    t5 = c² + b² = f² ; t6 = f² + a² = u² tu remarques que: a et u , sont impair mais t6 > t5 alors a serait > b et à c alors qu' il devrait être inferieur...de plus c,b et f étant pair, il s'agit d'un triplet multiple d'un primitif x ,y et z tel que o² - l² ; 2ol et o² + l² et un facteur K > 1

  22. #21
    invite35452583

    Re : petit problème

    Citation Envoyé par leg
    bonjour Matthias
    lorsque l'on repart de d qui n'est plus l'hypoténuse mais le côté du triangle rect t2, tu est d'accord que t2 > t1 et aussi que les nouveaux paramètres de t2 aussi, donc p > p' et p > q'; le deuxième côté de t2 est obligatoirement cselon l'hypthèse de départ, puisque d² + c² = u²;
    ("cn'est donc pas arbitraire dans le choix des trois entiers a, b et c.")
    tu en conviendras que 2p'q' = b est < 2pq = c.
    Justement, non !
    p est plus grand que p' et q' mais q peut-être petit.
    ainsi p' et q' peuvent être égaux à environ et le produit égal à environ à d/2.
    Tandis que p peut être environ égal à et le produit pq est égal à ce qui n'est pas nécessairement plus grand que d/2.

    Contre-exemple numérique construit avec 37 et 41 (deux premiers modulo 4 voisins)
    37.41=(6+i)(5+4i)=26²+29²=1517 (p'=26, q'=29)
    1517=37.41=p²-q² ; p+q=41 p-q=37 p=39 q=2 ; 1517=39²-2²
    p'q'=26.29=754 > pq=39.2 On a pourtant bien 39>29 et 39>26.
    Je suis d'accord 37.41 n'est pas un carré mais il suffit a priori de reprendre avec 37².41²
    Tout ce qui a à trouver c'est un nombre qui se décompose p²+q² avec p et q "proches" et se décompose comme produit m.n avec m et n "proches" ce qui implique que q est "très" petit. Ton inégalité p'q'<pq n'est alors plus vrai. Elle n'est vraie que pour les petits entiers car les décompositions sont "rares".

  23. #22
    leg

    Re : petit problème

    bonjour homotopie
    ton contre exemple n'est pas valable.il faut des triplets pythagoricien entiers donc des carrés parfaits ce qui est la supposition de départ .
    car effectivement si (p²+q²)² n'est pas un carré parfait alors ce n'est même plus la peine de chercher
    car a² +b² +c² =u² non parfait par ex 1517 ta valeur ,
    à quoi sert la supposition de matthias...?

    maintenant recommence avec des paramètres qui donne les triplets pytagoriciens, regarde ce que j'indique et prouve moi que:
    2p'q' = b est < 2pq = c, est faux dans les entiers naturels, soit , dans les triplets pythagoriciens!
    ou encore tu peux aussi me dire ou me montrer que la formule qui donne tous les triplets Py.. est fausse.

    et pour la forme, 37² + 41² = n qui n'est pas un carré parfait et pour cause ....je te laisse le soin de chercher un tout petit peu ....indication: il te faut deux nombres choisis dans un endroit bien précis....c'est facile

  24. #23
    leg

    Re : petit problème

    bonjour
    je viens de reprendre ton exemple homotopie, effectivement si au départ on prend 2 paramètres assez grand tel que p² + q² =d on a des chances de trouver p'² - q'² = p²+q² , tel que 2p'q' < 2pq et pas forcément superieur.ton exemple: (2 *26 *29) = b et (2*39*2)=c où c < b .
    avec mes excuses, et il me reste plus qu'à continuer sur cette piste;
    car peut être que la troisième paire de triplets
    a doit être > b et c alors qu'au départ il est inferieur; sauf bien sur si cet argument n'est pas valable.
    puisque b² + c² =f² d'où f² + a² = u²
    cas n°1: avec 26 et 29, a < B et c < b
    cas N° 2: je commence avec 39 et 2 , a > b et: ( c > b en principe)
    on sait que que b,c et f sont issus d'un triplet primitif (je ne sais pas si cela va servir) mais en supposant qu'on soit arrivé à: b, c et f et tel que f²+a² = u²
    il faut que f² < u² impérativement ce qui est juste mais il va falloir que u² - f² = or aest déjà connu il n'est plus question de retrouver 2 autres paramètres.
    et de plus a au départ est donné par p² -q² il faut maintenant que 2ol = a puisque o² - l² = f;
    c'est à dire que 2ol = p² - q²
    mais, comme un triplet multiple impose un primitif, avec des paramétres entiers inferieurs à i et h non entier, qui donnerait b,c et f pair.
    (exemple k =2, p=2 et q=1 donne le triplet 6,8 et 10 soit q = squart de 2 et p 2*squart de 2 si je ne me trompe pas ..) peut être qu'il y a là une contradiction a trouver...
    si tu as une idée..elle est bienvenue et merci pour ton intervention. ainsi que matthias. A+

  25. #24
    leg

    Re : petit problème

    suite :
    je pense que mon idée de grandeur et toujours la bonne piste.
    car même si j'augmente p et je diminue q pour obtenir c < b afin de contredir mon argument il ne faut pas perdre de vue que a et b ont été fixé et qu'ensuite il me faut donc retrouver a avec c qui est maintenant fixé par d ce qui me fait dire que si j'ai trouvé c trop petit je ne peux plus retrouver atel que a²+c² = f² afin que f² + b² = u²
    j'ai trouvé 26 et 29 puis 39 et 2 il me faut retrouver p''' et q''' tel que p'''²-q'''² = p² -q² = a ; et 2p"' q''' = 2p"q" = c.
    le premier triplet choisis, va conditionner toute la suite
    on sait au moins déjà, que si les triangles R sont construit de la sorte que t1<t2<t3...<tn; alors il n'y a pas de solution et on ne peut pas non plus faire l'inverse, descente infinie oblige.
    on sait que les 3 triangles R ayant le même hypoténuse uconstitue un bornage.
    y'à t'il une propriété de divisibilité entre ces différents triangles..?
    on sait aussi que si la solution existerait avec des triangles primitifs faisant appel à des paramètre non entiers alors il n'y a pas de solution. et d'ailleur on voit trés vite qu'en partant d'un triplet primitif on à obligatoirement un triplet multiple, pair. Ce qui exclu bien une solution générale de triplets primitif, sans utiliser un facteur k > 1 pour au moins le dernier triplet!
    d'autres idées...?

  26. #25
    leg

    Re : petit problème

    bonjour H et M, (vous n'avez pas d'idées ..ou est ce que la supposition de départ serait insoluble..?)

    La première question que j'aimerai savoir, faisant suite à l'idée de Homotopie, tu utilises les n de gauss, je suppose donc que tous les triplets Py.., peuvent s'ecrire sous cette forme, ainsi que 2 entiers naturels somme d'un entiers . ("car je ne connais pas ces nombres.")

    pour en revenir à cette supposition de départ,

    la troisième pair de triangle R, t5 et t6, soit b² + c² =f² puis f² + a² = u² ( le triplet b,c et f est pair) donc 2ol = f et non acar il est impair; si le deux paramètres qui donne le T.Py.. b, c et f sont i et h, alors:
    2ol = i² + h² = f.
    on sait que ce T.PY..est multiple d'un primitif en regardant et en essayant avec les paramètres de Homotopie 39 et 2 qui me donne c = 2*2*39 si le facteur K est 3 on retombe sur a = p² - q² = 26² et 29² = o² - l² = 13² et 2².
    ce qui me donnerait le triplet primitif: 165,52, et 173 or pour obtenir un t.PY.. pair et multiple il me faut un facteur K pair.
    Mais il faut aussi que j'ai a² +c² = E², = T3 afin d'obtenir le T4 donné par E² + b² = u²
    alors je reprend o et l (j'éssaye) tel que :
    165² + c² = F² ce qui est faux car c² = 156² je ne peux redéscendre les paramètres ....???
    qu'est ce que cela donnerait en entiers de Gauss, comme propriété...?

    On a 6 triangles Rect..qui doivent satisfaire la supposition de départ, donc 6 paires de paramètres avec des entiers, dont au moins un triplet Mulitiple, dont les paramètres entiers sont multiplier par un facteur K pair.

    par contre mille excuses a Homotopie, car je viens juste de voir ces 2 interventions du 27 06 à 12 H j'ai répondu trop vite à 19h a cause du match
    ta remarque sur l'ordre de grandeur des paramètres, pour un entier et trés juste.

    aurais tu idée ..? sans perdre de vue la formule des triplets Py..moi je cherche un raisonnement par l'absurde, avec la formule des T.PY..et le théorème de Pythagore.
    ("Mais peut être que cela n'est pas la bonne piste . pourtant pour résoudre le cas de la puissance N=4 ;dans le théorème de Fermat avec cette méthode c'était facile")
    A+

  27. #26
    leg

    Re : petit problème

    En définitive c'est E et F qui pose probléme .
    a² + c² = E² et u² - b² = E², avec uniquement E non entier; et il en est de même pour B² - c² = F² d'où
    U² - a² = F² où F n'est pas un entier .

    alors si troutes les solutions pythagoriques entiéres peuvent s'écrire avec les entiers de Gauss et que la supposition de départ n'à pas de solution, elle n'en a pas non plus dans ces nombres!

    on à l'impression de retrouver le théorème de Fermat, si cette supposition en serait une suite, alors il n'y a pas de solution ou encore si on peut la résumer à la même expréssion, que celle qui à permis à A willes de résoudre ce théorème pas de solution( les équation de Fermat correspondent à B² =.....etc ).

    donc on peut dire que cette supposition de départ ne peut en aucun cas avoir un Triplet pythagoricien x,y et z contenant deux produits de puissances N >= 2 avec N pair sinon le théorème de Fermat serait faux .
    question:
    est ce que l'on pourrait montrer justement que cette supposition pourrait en avoir?

  28. #27
    invite35452583

    Re : petit problème

    Citation Envoyé par leg
    La première question que j'aimerai savoir, faisant suite à l'idée de Homotopie, tu utilises les n de gauss, je suppose donc que tous les triplets Py.., peuvent s'ecrire sous cette forme, ainsi que 2 entiers naturels somme d'un entiers . ("car je ne connais pas ces nombres.")
    Les entiers de Gauss Z[i] sont les complexes que l'on peut écrire sous la forme m+in avec m et n entiers relatifs.
    Ils sont très utiles (généralement en tout cas) quand on travaille sur les sommes de carrés car on a : (m+in)(m-in)=m²+n² où m-in est le conjugué de m+in.
    Pour la paramétrisation classique, on peut l'exprimer ainsi en entiers de Gauss :
    (p²-q²)²+(2pq)²=[(p²-q²)+2ipq][(p²-q²)-2ipq]=(p+iq)²(p-iq)²
    (p²+q²)²=[(p+iq)(p-iq)]²
    d'où en posant z=p+iq et z*=p-iq (le conjugué)
    (zz*)²=z².(z*)²=z²(z²)*
    Jusqu'ici une simple réécriture. Mais l'avantage des entiers de Gauss c'est la multiplication (ça peut s'exprimer sans mais c'est plus confortable)
    Comment écrire de manières différentes en sommes de carrés un même nombre ?
    une somme de carrés c'est un produit de la forme zz*.
    Ainsi par exemple si z=z1z2, on a alors
    (z1z2)(z1z2)*=(z1z2*)(z1z2*) (Les autres retournent sur les deux précédents)
    Exemple [(2+i)(3+2i)].conjugué=[(2+i)(3-2i)].conjugué
    (4+7i).conjugué=(8-i).conjugué
    4²+7²=8²+1²=65=5.13=(2+i)(2-i) . (3+2i)(3-2i)
    On veut que la somme des carrés soient le carré d'un entier, donc de la forme (zz*)²
    Et on veut qu'on puisse l'écrire de manières différentes, il suffit que z soit un produit.
    z=z1z2
    (zz*)²=
    i) (z1z1*z2z2*).conjugué =entier au carré car z1z*z2z2* est son propre conjugué donc réel et donc entier.
    ii) (z1²z2z2*).conj
    iii) (z1z1*z2²).conj
    iv) (z1²z2²).conj
    v) (z1²z2*²).conj
    Exemple numérique (déjà vu en ajoutant l'oublié)
    4225=65²=52²+39²=60²+25²=56²+3 3²=63²+16²
    Et en termes de paramètres ?
    [(p+iq)(p-iq)]²=(p+iq)²(p-iq)² chez les "primitifs" c'est "i)=iv)" et "i)=v)"
    Les ii) et iii) ne sont pas primifs ils sont des multiples :
    z2z2* fois (z1z1*)²=z1²z1*² pour le ii)
    z1z1* fois (z2z2*)²=z2²z2*² pour le iii)

    Citation Envoyé par leg
    bonjour H et M, (vous n'avez pas d'idées ..ou est ce que la supposition de départ serait insoluble..?)
    En gros pas trop d'idées
    solution de départ insoluble ???
    désolé

    Le problème ici est que l'on a :
    u²=(a+id)(a-id)=(b+ie)(b-ie)=(c+if)(c-if) ce genre de décompositions ça se traite bien dans les entiers de Gauss.
    Mais a²+b²=f², par exemple, n'a plus d'écriture "naturelle". Il semble donc que la résolution (si elle existe ) ne soit pas arithmétique.
    Un mélange d'arithmique et de numérique basée sur ton idée : ???? 6 couples de paramètres, c'est trop pour moi dans un domaine où je ne suis pas très intuitif.

    Bon courage

  29. #28
    leg

    Re : petit problème

    si je t'ai bien suivi, il parait donc évident que si il y avait une solution elle pourait s'écrire avec les nombres complexes ,d'où l'absence de solution dans les entiers implique pas de solution en entiers de Gauss.

    l'idée que je crois avoir trouvé:
    (" en espérant que comme la première fois, tu ne vas pas me trouver un contre exemple aussi éfficace ... " si tel est le cas, je t'en remercirai car celà fait avancer le schmilblik.)
    je reste donc dans la formule des T.Py et Pythagore.

    on a p différent de p' différent de p" ainsi que p"' entiers
    ainsi que pour q ..q'..q"..q"' entiers, positifs
    T.P , veut dire Triplet pytagoricien donné par la formule des T.P.

    on part du premier T.P :
    A² + B² = D² ,et D² + C² = U² ,ce qui se décompose:

    ....A².....+....B²....=....D²
    (p²-q²)² + (2pq)² = (p²+q²)² qui donne:
    [((p²-q²)² + (2pq)²) + (2p'q')² = (p'² +q'²)²] = (p'²-q'²)²+(2p'q')² = (p'² +q'²)²
    nous avons deux triangles rectangles différents, puis une infinité ce qui ne pose aucune contre indication. suite:
    je reprend A² = p²-q² = p"² -q"².

    (p"² -q"²)² + [(2p"q")²=(2p'q')²] = (p"² + q"²)² = (p"'² -q"'²)² et (p"'²-q"'²)² + [(2pq)²=(2p"'q"')²]=(p"'²+q"'²)² =
    (p'² +q'²) = U².
    si je supprime à droite p"' et q"' il me reste :
    (p"²+q"²)² + (2pq)² = (p'²+q'²)²
    c'est à dire un T.P donné avec p différent de p' et de p" ainsi que pour q différent de q' et de q"
    si et seulement si, je ne sais pas qu'il pourrait exister p"' et q"'. ce qui me ramène au début....

  30. #29
    leg

    Re : petit problème

    Citation Envoyé par homotopie Voir le message
    Justement, non !
    p est plus grand que p' et q' mais q peut-être petit.
    ainsi p' et q' peuvent être égaux à environ et le produit égal à environ à d/2.
    Tandis que p peut être environ égal à et le produit pq est égal à ce qui n'est pas nécessairement plus grand que d/2.

    Contre-exemple numérique construit avec 37 et 41 (deux premiers modulo 4 voisins)
    37.41=(6+i)(5+4i)=26²+29²=1517 (p'=26, q'=29)
    1517=37.41=p²-q² ; p+q=41 p-q=37 p=39 q=2 ; 1517=39²-2²
    p'q'=26.29=754 > pq=39.2 On a pourtant bien 39>29 et 39>26.
    Je suis d'accord 37.41 n'est pas un carré mais il suffit a priori de reprendre avec 37².41²
    Tout ce qui a à trouver c'est un nombre qui se décompose p²+q² avec p et q "proches" et se décompose comme produit m.n avec m et n "proches" ce qui implique que q est "très" petit. Ton inégalité p'q'<pq n'est alors plus vrai. Elle n'est vraie que pour les petits entiers car les décompositions sont "rares".
    bonjour .
    Matthias , Homotopie
    je reviens sur ce problème.
    la contradiction a mon hypotèse faite par Homotopie étant montrée, il reste peut être une autre solution:

    au lieu de partir en escargot, je part donc d'une solution plus grande afin que B soit > c, pour éviter mon hypothèse contradictoire (post pércédent)

    or en démontrant qu'un triplet pythagoricien primitif, donné avec des entiers non nul et premiers entre eux, le seul triplet qui pose problème est le triplet primitif
    B, c et F trois entiers pairs qui vérifie B² + c² = f²il existe donc un couple de paramètre p" et q" entiers premiers entre eux et un facteur K >1!
    qui donne ce triplet.

    il semblerait que la limite pour paramétrer le triplet B,c et f, viennent justement du premier triangle T1=
    a² + b² = d² avec p =29 et q =26; où b =1908

    T4 = est donné par p' et q' = 39 et 2
    d'² + c² = u²; où c = 156

    T3 = serra donc donné par: K , p" et q"
    b'² + c'² = f² trois entiers pairs qui sont divisible par k

    pour cet exemple il est impossible de paramètrer le triplet b', c' et f ! f ne peut être un entier.
    1908² + 156² = ?²

    sachant qu'un triplet primitif ou multiple d'un primitif ne peut être donné par p et q entiers non nul, premiers entre eux; ou par p" et q" entiers non nul premiers entre eux et un facteur K >1 entiers naturel!
    cette solution à ce petit problème se résoud avec les entiers naturels!
    car en effet deux réels ne peuvent donner un triplet primitif ou multiple d'un primitif qui ne saurait donné par p et q tel que défini, ci dessus, ou avec un facteur K .(ce qui se démontre!).

    je pense qu'il est impossible de paramétrer le triplet pair du triangle T3! car il dépend de T1 et T4; où T3 <T1 <T4

    alors que les paramétres de T1 sont > à ceux de T4.

    j'espère que cela vous donnera une idée! A+ ,je continue.

  31. #30
    leg

    Re : petit problème

    Citation Envoyé par leg Voir le message
    bonjour .
    Matthias , Homotopie
    je reviens sur ce problème.
    il semblerait que la limite pour paramétrer le triplet B,c et f, viennent justement du premier triangle T1=
    a² + b² = d² avec p =29 et q =26; où b =1508 et non1908 ,
    T4 = est donné par p' et q' = 39 et 2
    d'² + c² = u²; où c = 156

    T3 = serra donc donné par: K , p" et q"
    b'² + c'² = f² trois entiers pairs qui sont divisible par k

    pour cet exemple il est impossible de paramètrer le triplet b', c' et f ! f ne peut être un entier.
    1508² + 156² = ?²

    .
    le triplet b',c' et f si il existe ,
    il correspond à:
    (2pq)² + (2p'q')² = (k u²+v²)²

    ce qui donne:
    4 (pq)² + 4 (p'q')² = (k u²+v²)²

    où : 2pq= 1508, 2p'q' = 156 et k u²+v² = f
    on peut dire que K = 4 au minimum, ("et u et v sont deux entiers premiers entre eux");
    puisqu'il doit diviser 2pq ainsi que 2p'q'.

    de plus le triplet b',c' et f est un multiple d'un primitif.
    donnant la solution :b'²+c'² = f'²
    il s'écrit:
    (k u² - v²)² + (k 2uv)² = (k u² + v²)²

    où:

    4 (pq)² = (k u² - v²)²
    4 (p'q')² = (k 2uv)²
    K est donc un multiple de 4

    du fait qu'il existe un triplet primitif
    u² -v² =x, 2uv = y et u²+v² = z

    ce que je trouve curieux et qui pourrait être une piste,
    1508 et 156 sont divisible par 4 ce qui est obligatoire!

    mais ce qui ne l'est moins j'obtiens deux nombres impairs
    si j'avais obtenus deux nombre pairs, il existerait une autre solution primitive inferieur ..etc
    alors supposons que l'on obtienne soit deux impairs ou 2 pairs il n'exsite pas de solution au problème! en effet un triplet primitif donnant b', c' et f est constitué de deux impairs et un pair = 2uv!

    avez vous des idées.

Page 1 sur 2 1 DernièreDernière

Discussions similaires

  1. Un petit problème qui me pause problème lol
    Par invitef2853e5d dans le forum Mathématiques du supérieur
    Réponses: 13
    Dernier message: 31/03/2009, 15h28
  2. Petit problème,
    Par ClaudeH dans le forum Science ludique : la science en s'amusant
    Réponses: 4
    Dernier message: 12/06/2007, 14h24
  3. Petit problème sur un problème ^^
    Par herman dans le forum Physique
    Réponses: 13
    Dernier message: 22/05/2007, 17h02
  4. Petit problème
    Par invite88a78667 dans le forum Chimie
    Réponses: 0
    Dernier message: 04/12/2006, 19h52
  5. Petit problème
    Par invite69f268fb dans le forum Mathématiques du supérieur
    Réponses: 12
    Dernier message: 05/03/2004, 06h36